0
$\begingroup$

A 1D finite square well is generally defined either by \begin{equation} V(x)=\begin{cases} 0\qquad -a\leqslant x\leqslant a\\ V_0\qquad \text{otherwise} \end{cases} \tag{1} \end{equation} or \begin{equation} V(x)=\begin{cases} -V_0\qquad -a\leqslant x\leqslant a\\ 0\qquad \text{otherwise} \end{cases} \tag{2} \end{equation}

Where $a>0$ and $V_0>0$. It is just a matter of defining the zero of your potential.

Infinite square well can be regarded as the limit $V_0\to+\infty$ of $(1)$, i.e.

\begin{equation} V(x)=\begin{cases} 0\qquad -a\leqslant x\leqslant a\\ +\infty\qquad \text{otherwise} \end{cases} \tag{3} \end{equation}

In this case the wavefunction is zero outside the well and you solve Schrödinger equation inside applying boundary conditions.

What about taking the limit of $(2)$? \begin{equation} V(x)=\begin{cases} -\infty\qquad -a\leqslant x\leqslant a\\ 0\qquad \text{otherwise} \end{cases} \tag{4} \end{equation}

Since we are now dealing with infinities I'm not sure whether saying $(3)$ and $(4)$ only differ for the choice of the of the potential. Also, what can I say about the wave function inside and outside in case $(4)$?

$\endgroup$
1
  • $\begingroup$ Intuitively I'd say that the Hamiltonian becomes unbounded from below, no? That being said, the infinite square-well is properly modeled using the corresponding boundary conditions on the domain of the Hamiltonian, for example... $\endgroup$ Commented Nov 12 at 22:40

1 Answer 1

1
$\begingroup$

A good way to approach problems with "infinities" is to study/solve them for finite values and then take an appropriate limit. So, here, you could write $$V_\infty(x) = \lim_{V_0\to\infty} V_{V_0}(x) = \lim_{V_0\to\infty} \begin{cases} 0 & x \in [-a,a] \\ V_0 &\text{otherwise} \end{cases}$$ and solve the Schrödinger equation, and then take the limit.

This implies that you can carry over the properties of the solution of the Schrödinger equation for a non-infinite $V_0$. In particular the independence of the choice of the zero of the potential (e.g. the invariance under arbitrary shifts $V(x) \to V(x) + \mathrm{const}$), implying that (3) and (4) are equivalent. If you study the wavefunctions carefully, you will see that they acquire a constant phase as you do the potential shift. But this is phase is not observable anyway (recall that states are actually elements of a projective Hilbert space). Any observable properties will remain invariant under this potential shift, in particular the (non-) vanishing of the intensity $|\psi|^2$ inside/outside the well.

$\endgroup$
1
  • $\begingroup$ If $(3)$ and $(4)$ are equivalent, then to study $(4)$ without passing through the limit, you would say that the wavefunction must be zero outside the well just as in $(3)$. Is that correct? $\endgroup$ Commented Nov 26, 2021 at 8:10

Your Answer

By clicking “Post Your Answer”, you agree to our terms of service and acknowledge you have read our privacy policy.

Not the answer you're looking for? Browse other questions tagged or ask your own question.